The number line shows the car's starting point and the ending point.
Find the distance between them.

The Number Line Shows The Car's Starting Point And The Ending Point.Find The Distance Between Them.

Answers

Answer 1

Answer:

-12/7

Step-by-step explanation:

Answer 2

[tex] \frac{ - 5}{7} - \frac{7}{7} [/tex]

[tex] \frac{ - 5 - 7}{7} [/tex]

[tex] \frac{ - 12}{7} [/tex]

Third option is the answer.


Related Questions

log8-log4 ÷ log4-log2=





Answers

The answer is log(4)-1

For the function G defined by G(x)=5x+3, find G(r+5)

Answers

Given function:

g(x) = 5x + 3

Find

g(r+5)

Substitute x with r = 5:

g(r + 5) = 5(r + 5) + 3 = 5r + 25 + 3 = 5r + 28

Answer:

G ( r + 5 ) = 5r + 28

Step-by-step explanation:

Given ;

G ( x ) = 5x + 3

To Find :-

G ( r + 5 )

Solution :-

plug r + 5 as x in the function.

G ( r + 5 ) = 5 ( r + 5 ) + 3

distribute 5

G ( r + 5 ) = 5r + 25 + 3

combine like terms

G ( r + 5 ) = 5r + 28

Find the value of y

Help please

Answers

Answer:

6

Step-by-step explanation:

Set your formula up as

15 = 2y+3

15 - 3 = 2y

12 = 2y

12 / 2 = y

6 = y

plz help me with this math and also explain

Answers

Step-by-step explanation:

[1]

SI = $250Rate (R) = 12[tex] \sf \dfrac{1}{2}[/tex] %Time (t) = 4 years

[tex]\longrightarrow \tt { SI = \dfrac{PRT}{100} } \\ [/tex]

[tex]\longrightarrow \tt { 250 = \dfrac{P \times 12\cfrac{1}{2} \times 4}{100} } \\ [/tex]

[tex]\longrightarrow \tt { 250 = \dfrac{P \times \cfrac{25}{2} \times 4}{100} } \\ [/tex]

[tex]\longrightarrow \tt { 250 = \dfrac{P \times 25 \times 2}{100} } \\ [/tex]

[tex]\longrightarrow \tt { 250 = \dfrac{P \times 50}{100} } \\ [/tex]

[tex]\longrightarrow \tt { 250 \times 100 = P \times 50} \\ [/tex]

[tex]\longrightarrow \tt { 25000 = P \times 50} \\ [/tex]

[tex]\longrightarrow \tt { \dfrac{25000}{50} = P } \\ [/tex]

[tex]\longrightarrow \underline{\boxed{ \green{ \tt { \$ \; 500 = P }}}} \\ [/tex]

Therefore principal is $500.

__________________

[2]

2/7 of the balls are red.3/5 of the balls are blue.Rest are yellow.Number of yellow balls = 36

Let the total number of balls be x.

→ Red balls + Blue balls + Yellow balls = Total number of balls

[tex]\longrightarrow \tt{ \dfrac{2}{7}x + \dfrac{3}{5}x + 36 = x} \\ [/tex]

[tex]\longrightarrow \tt{ \dfrac{10x + 21x + 1260}{35} = x} \\ [/tex]

[tex]\longrightarrow \tt{ \dfrac{31x + 1260}{35} = x} \\ [/tex]

[tex]\longrightarrow \tt{ 31x + 1260= 35x} \\ [/tex]

[tex]\longrightarrow \tt{ 1260= 35x-31x} \\ [/tex]

[tex]\longrightarrow \tt{ 1260= 4x} \\ [/tex]

[tex]\longrightarrow \tt{ \dfrac{1260 }{4}= x} \\ [/tex]

[tex]\longrightarrow \underline{\boxed{ \tt { 315 = x }}} \\ [/tex]

Total number of balls is 315.

A/Q,

3/5 of the balls are blue.

[tex]\longrightarrow \tt{ Balls_{(Blue)} =\dfrac{3 }{5}x} \\ [/tex]

[tex]\longrightarrow \tt{ Balls_{(Blue)} =\dfrac{3 }{5}(315)} \\ [/tex]

[tex]\longrightarrow \tt{ Balls_{(Blue)} = 3(63)} \\ [/tex]

[tex]\longrightarrow \underline{\boxed{ \green {\tt { Balls_{(Blue)} = 189 }}}} \\ [/tex]

One leg of a right triangle is 7 inches longer than the other leg, and the hypotenuse is 35 inches. Find the lengths of the legs of the triangle.

Answers

Answer: 21, 28

Step-by-step explanation:

Side #1 = xSide #2 = x + 7Hypotenuse = 35

Use the Pythagorean Theorem [tex]a^{2}+b^{2}=c^{2}[/tex]:

a = xb = x + 7c = 35

Substitute in the values & solve:

[tex]x^{2}+(x+7)^{2}=35^{2}\\x^{2}+x^{2}+14x+49=1225\\2x^{2}+14x+49-1225=0\\2x^{2}+14x-1176=0\\2(x^{2}+7x-588)=0\\2(x + 28)(x - 21)=0\\x_{1}=-28, x_{2}=21[/tex]

-28 is not a possible solution since you can't have negative inches...

a = x = 21b = x + 7 = 21 + 7 = 28c = 35

Arvin has $10000 to invest. He invests part in a term deposit paying 5%/year, and the remainder in Canada savings bonds paying 3.4%/year. At the end of the year, he earned simpler interest of $413. How much did he invest at 5%/year?

Answers

Answer:

$4,562.5

Step-by-step explanation:

The amount Arvin has to invest, P = $10,000

The interest paid on the investment in the term deposit = 5%/year

The interest paid om the investment in Canada savings bonds = 3.4%/year

The amount Arvin earned at the of the year as simple interest, A = $413

Let, x, represent the amount Arvin invested in the term deposit and let, y, represent the amount he invested in Canada savings bonds, we can get the following system of equations

x + y = 10,000...(1)

0.05·x + 0.034·y = 413...(2)

Making y the subject of equation (1) and substituting the value in equation (2), we get;

From equation (1), we get, y = 10,000 - x

Plugging the above value of y in equation (2) gives;

0.05·x + 0.034 × (10,000 - x) = 413

∴ 0.05·x - 0.034·x + 340 = 413

x = (413 - 340)/(0.05 - 0.034) = 4,562.5

Therefore, the amount Arvin invested in the term deposit at 5%, x = $4,562.5

(y = 10,000 - x

∴ y = 10,000 - 4,562.5 = 5,437.5

The amount Arvin invested in Canada savings bonds, y = $5,437.5.)

Find the measure of the missing angle using the exterior angle sum theorm.

Answers

Answer:

85°

Step-by-step explanation:

The exterior angle of a triangle is=sum of the opposite interior angles

So

? °=45°+40°

Which point is in the solution set of this system inequalities?

A. (0,0)

B. None of these

C. (5,1)

D. (3,7)

Answers

Answer:

B

Step-by-step explanation:

To find which ordered pairs are solutions to the inequalities we can simply plug in the x and y values of the ordered pairs into the inequalities and if the equation is true for both inequalities then the ordered pair is a solution to the inequalities.

For (0,0)

x = 0

y = 0

y > x + 5

Substitute 0 for y and x

0 > 0 + 5

Simplify right side

0 > 5

The inequality is not true as 5 is greater than 0, not less than. So immediately we can eliminate answer choice A.

For (5,1).

x = 5

y = 1

y > x + 5

Substitute 5 for x and 1 for y

1 > 5 + 5

Simplify right side

1 > 10

Again, the equation is not true as 1 is not greater than 10. This means that c cannot be the answer

For (3,7)

x = 3

y = 7

y > x + 5

Substitute 3 for x and y for 7

7 > 3 + 5

Simplify right side

7 > 8

7 is not greater than 8 meaning that (3,7) cannot be a solution to the inequalities

None of the ordered pairs created true equations hence the answer is B

if f(x) = 2x/7 +4, which of the following is the inverse of f(x)?

Answers

Answer:

The answer is C

Step-by-step explanation:

Switch x and y

[tex]x = \frac{2y}{7} + 4[/tex]

Solve for y

[tex]y = \frac{7(x - 4)}{2} [/tex]

help help help pls :)

Answers

Answer:

[tex]opposite\approx 70.02[/tex]

Step-by-step explanation:

The triangle in the given problem is a right triangle, as the tower forms a right angle with the ground. This means that one can use the right angle trigonometric ratios to solve this problem. The right angle trigonometric ratios are as follows;

[tex]sin(\theta)=\frac{opposite}{hypotenuse}\\\\cos(\theta)=\frac{adjacent}{hypotenuse}\\\\tan(\theta)=\frac{opposite}{adjacent}[/tex]

Please note that the names ([tex]opposite[/tex]) and ([tex]adjacent[/tex]) are subjective and change depending on the angle one uses in the ratio. However the name ([tex]hypotenuse[/tex]) refers to the side opposite the right angle, and thus it doesn't change depending on the reference angle.

In this problem, one is given an angle with the measure of (35) degrees, and the length of the side adjacent to this angle. One is asked to find the length of the side opposite the (35) degree angle. To achieve this, one can use the tangent ([tex]tan[/tex]) ratio.

[tex]tan(\theta)=\frac{opposite}{adjacent}[/tex]

Substitute,

[tex]tan(35)=\frac{opposite}{100}[/tex]

Inverse operations,

[tex]tan(35)=\frac{opposite}{100}[/tex]

[tex]100(tan(35))=opposite[/tex]

Simplify,

[tex]100(tan(35))=opposite[/tex]

[tex]70.02\approx opposite[/tex]

solve the simultaneous equation: x-y=2
xy=36​

Answers

Answer:

Y= –7.08, 5.08

Step-by-step explanation:

hope ya ready bro.

X=36/Y

replace 36/y instead of X

36/Y–Y=2===> 36–Y²=2Y===> Y²+2Y–36=0

Y1= –1+√37≈ –7.08

Y2= –1–√37≈ 5.08

one more question
(-8)+___=-17

Answers

Answer:

-9

Step-by-step explanation:

Answer:

-9

Step-by-step explanation:

-8 +___= - 17

___=-17 +8

___=-9

=-9

Someone please help me ASAP

Answers

Step-by-step explanation:

a vector multiplied by a scalar is equal to it's image. The expression above gives an equation and after solving, it gives you the image

In In 5x + In In (x - 1) = 2

Answers

Answer:

exact form: x=-1/2

decimal form: x=-0.5

On a coordinate plane, a line goes through (negative 3, negative 4) and (3, 0).
What are the necessary criteria for a line to be perpendicular to the given line and have the same y-intercept?

The slope is Negative three-halves and contains the point (0, 2).
The slope is Negative two-thirds and contains the point (0, −2).
The slope is Three-halves and contains the point (0, 2).
The slope is Negative three-halves and contains the point (0, −2

Answers

Answer:

The slope is Negative three-halves and contains the point (0, 2).

Step-by-step explanation:

(-3,-4)(3,0)

M= -4/-6 = 2/3

⊥M = -3/2

0 = 2/3(3) + B

B=2

P(x) is a polynomial. here are a few values of p(x).
P(-5) = - 2
P(-3) = 6
P(3) = 7
P(5) = -1
What is the remainder when P(x) is divided by (x+5)?
What is the remainder when P(x) is divided by (x-3)?

Answers

Given:

Values of a polynomial P(x).

[tex]P(-5)=-2[/tex]

[tex]P(-3)=6[/tex]

[tex]P(3)=7[/tex]

[tex]P(5)=-1[/tex]

To find:

The remainder when P(x) is divided by (x+5).

The remainder when P(x) is divided by (x-3).

Solution:

If a polynomial P(x) is divided by (x-a), then the remainder is P(a).

If the polynomial P(x) is divided by (x+5), then the remainder is P(-5).

[tex]P(-5)=-2[/tex]

Therefore, the remainder is -2 when P(x) is divided by (x+5).

If the polynomial P(x) is divided by (x-3), then the remainder is P(3).

[tex]P(3)=7[/tex]

Therefore, the remainder is 7 when P(x) is divided by (x-3).

a^2×c^2/c^2×d^2+bc/ad reduce the algebraic ​

Answers

Answer:

[tex]\frac{a^2*c^2}{c^2*d^2}+\frac{bc}{ad}= \frac{a^3 + bcd}{ad^2}[/tex]

Step-by-step explanation:

Given

[tex]\frac{a^2*c^2}{c^2*d^2}+\frac{bc}{ad}[/tex]

Required

Simplify

We have:

[tex]\frac{a^2*c^2}{c^2*d^2}+\frac{bc}{ad}[/tex]

Cancel out [tex]c^2[/tex]

[tex]\frac{a^2*c^2}{c^2*d^2}+\frac{bc}{ad}= \frac{a^2}{d^2}+\frac{bc}{ad}[/tex]

Take LCM

[tex]\frac{a^2*c^2}{c^2*d^2}+\frac{bc}{ad}= \frac{a^3 + bcd}{ad^2}[/tex]

If
5
3 cosα = in the first quadrant, what does cot α
equal?

Answers

Answer:

5/4

Step-by-step explanation:

To Find :-

cot a .

Solution :-

By question ,

=> cos a = 5/3 = b/h

=> p = √ 5² - 3² = √ 25 -9 = 4

Therefore ,

=> cot a = b/p = 5/4

In which section of the number line is 32−−√?

Answers

where's the number line?

maybe u can attach it at the comments:)

Answer:

Section B

Step-by-step explanation:

find the volume of the rectangular prism. plz answer this lol

Answers

Answer:

.....how when the dimensions are not even clear lol

Answer:

48 cm³

Step-by-step explanation:

the volume of a rectangular prism= length × breadth × height

= 8× 3 × 2

= 48 cm³

The equation d = m/v can be used to calculate the density,d, of an object with mass,m, and volume,V. Which is an equivalent equation solver for V?

Answers

Answer:

v = m/d

Step-by-step explanation:

since

[tex]d = \frac{m}{v} \: \\ \\ v = \frac{m}{d} [/tex]

Please help. I don’t understand I rlly appreciate it if you help!

Answers

Hi there!

We know that the angles ∠B and ∠A are supplementary because they are both interior angles. Therefore:

180 = ∠A + ∠B

Rewrite:

180 = (6x - 48) + (4x + 38)

Combine like terms:

180 = 10x - 10

Solve for x:

190 = 10x

x = 19

Solve for ∠B by plugging in this value of x:

∠B = 4(19) + 38

∠B = 76 + 38

∠B = 114

What is the slope of the line? What is the y-intercept of the line? y = -3x + 4

Answers

Answer:

slope= -3/1

y-intercept= 4

Answer:

m = -3

y intercept = 4

Step-by-step explanation:

The given equation of the line is ,

[tex]\implies y = -3x+4[/tex]

We know that the Standard equation of Slope Intercept Form of the line is,

[tex]\implies y = mx + c[/tex]

Where ,

m is slope c is y intercept

On comparing to the Standard form of the line we get ,

[tex]\implies Slope = -3 [/tex]

[tex]\implies y - intercept= 4 [/tex]

a 10 foot ladder rests against a vertical wall if the bottom of the ladder slides away from the wall at a speed of 2 ft/s how fast is the angle betwween the top of the ladder and the wall changing when that angle is

Answers

Answer:

d∅/dt = √2/5 Rad/sec

Step-by-step explanation:

According to the Question,

Given That, a 10-foot ladder rests against a vertical wall if the bottom of the ladder slides away from the wall at a speed of 2 ft/s how fast is the angle between the top of the ladder and the wall changing when that angle is π/4.

Solution,

Let x be the Distance between the base of the wall and the bottom of the ladder.

and let ∅ be the angle between the top of the ladder and the wall.

Then, Sin∅ =x/10  so, x=sin∅ *10

Differentiating with respect to time t we get,

dx/dt = 10 * cos∅ * d∅ /dt

We have given that dx/dt = 2 ft/s and ∅ =π/4

Now, Put these value we get

2 = 10 *(cos(π/4))* d∅/dt

2 = 10/√2 * d∅/dt

d∅/dt = √2/5 Rad/sec

What is 6.273 rounded to the nearest thousandths?

Answers

Answer:

6.270

Step-by-step explanation:

3 is below 5 so you just turn it into a 0. If it was 5 or above you would just add a number to the 7.

A number ending in ___ is never a perfect square. ​

Answers

Answer:

2, 3, 7 or 8

Step-by-step explanation:

The sum of three consecutive odd numbers is 63. What are the numbers?​

Answers

19,21,23. Adding all of these consecutive odd numbers is equal to 63

Answer:

19, 21 and 23

Step-by-step explanation:

→ Make an algebraic expression for the 3 consecutive numbers

2x + 1, 2x + 3 and 2x + 5

→ Add the expressions together and make it equal to 63

2x + 1 + 2x + 3 + 2x + 5 = 63

→ Simplify

6x + 9 = 63

→ Minus 9 from both sides

6x = 54

→ Divide both sides by 6

x = 9

→ Resubstitute x = 9 into the 3 expressions

x = 9 into 2x + 1 is 19

x = 9 into 2x + 3 is 21

x = 9 into 2x + 5 is 23

Identify a horizontal or vertical stretch or compression of the function by observing the equation of the function .

Answers

Bdbxbcjncnxndnsnns Identify a horizontal or vertical stretch or compression of the function by observing the equation of the function .hdbdbebqb

sue has 18 pieces of candy
tony has 18 pieces of candy
sue then gives some to tony
sue then eats five of hers
tony eats half of his
write the expressions for the number of pieces candy sue and tony now have?

Answers

Answer:

Sue candy = 13 - x

Tony candy = 9 + 1/2x

Step-by-step explanation:

Sue candy = 18

Tony candy = 18

Let x = some candy gives to tony

Sue candy = 18 - x

Tony candy = 18 + x

sue then eats five of hers

Sue candy = 18 - x - 5

= 13 - x

tony eats half of his

Tony candy = 1/2(18 + x)

= 18/2 + x/2

= 9 + 1/2x

Expressions for the number of pieces candy sue and tony now have:

Sue candy = 13 - x

Tony candy = 9 + 1/2x

someone help me for this algebra task please

Answers

Answer:

200

Step-by-step explanation:

Substitute 15 for y

[tex] \frac{1}{5} x - \frac{2}{3} (15) = 30[/tex]

[tex] \frac{1}{5} x - 10 = 30[/tex]

[tex] \frac{1}{5} x = 40[/tex]

[tex]x = 200[/tex]

Other Questions
While some property owners choose to perform both the property and asset manager functions themselves, many commercial property owners choose to employ professional property managers instead. The property manager works under a management contract in which the manager is empowered to serve as the owner's fiduciary. This type of relationship is more commonly referred to as a(n) Can someone please help me by putting me where each one goes? !! The following statement is incorrect as stated "Pesticides create pesticide resistance in mosquitoes." Rewrite this sentence so that it is a more accurate statement of what actually occurs when pesticides are used. Why is this concept important to know in trying to control outbreaks of mosquito transmitted diseases such as eastern equine encephalitis (also known as Triple E)? Which statement best completes the diagram? Consider a relation on the set of all states in the United States given by: two states are related if they have a border in common. Is it an equivalence relation How to find the frequency in a histogram Find the time required for an investment of 5000 dollars to grow to 8600 dollars at an interest rate of 7.5 percent per year, compounded quarterly. Would you prefer to use an integrated router, switch, and firewall configuration for a home network, or would you prefer to operate them as separate systems If 2 percent growth is your break-even point for an investment project, under which outlook for the economy would you be more inclined to go ahead with the investment: (1) A forecast for economic growth that ranges from 0 to 4 percent, or (2) a forecast of 2 percent growth for sure, assuming the forecasts are equally reliable? What core principle does this illustrate? what's this problem please You want to put a 5 inch thick layer of topsoil for a new 23 ft by 27 ft garden. The dirt store sells by the cubic yard. How many cubic yards will you need to order? Convert 45 minutes to seconds. There are seconds in 45 minutes (Simplify your answer.) how many seconds are in 45 minutes When the economy is working properly, what is the acceptable range of unemployment? a 7-10% b 12-15% c 4-6% d 0-3%When a nation imports more than it exports, economists say it has which of the following? a a trade deficit b a balance of trade c a national defense d a trade surplusA high level of energy consumption indicates a a large labor force working mainly in subsistence agriculture. b a high level of economic development. c a population with a low level of education. d a high infant mortality rate For the expression x-5, what would the value be if x=18? What is the mass of 2.30x10^22 formula units of NaOH (molar mass =40.0g/mol) What is the area of this figure Help please!!!! Why is it important to eliminate debt as soon as possible? if you are good at graphs this is good but please help it would mean a lot, I will give brain thingy (Will put as Brainliest) Please help! :")A speciality candy shop makes chocolate covered cherry and graham cookie bites. The cherry is spherical with a diameter of 3cm and the graham cookie is a rectangular prism with a base measuring 5cm by 5cm and a thickness of 0.5cm. What is the total surface area of the bite if each piece is drenched in chocolate from top to bottom before being put together, to the nearest tenth of a square centimetre. What were the advantages for Mendel in using pea plants for his breeding experiments?